silver (unfinished gist)
Some checks failed
Build latex and deploy / checkout (push) Failing after 15m15s

This commit is contained in:
Josia Pietsch 2024-02-14 02:06:55 +01:00
parent ebdf929dd7
commit deaf6a6bb6
Signed by: josia
GPG key ID: E70B571D66986A2D
6 changed files with 180 additions and 36 deletions

View file

@ -9,7 +9,7 @@ Fix an ordinal $\beta$.
We recursively define We recursively define
\begin{IEEEeqnarray*}{rClClr} \begin{IEEEeqnarray*}{rClClr}
\beta &+& 0 &\coloneqq & \beta\\ \beta &+& 0 &\coloneqq & \beta\\
\beta &+& (\alpha + 1)&\coloneqq &(\beta + \alpha),\\ \beta &+& (\alpha + 1)&\coloneqq &(\beta + \alpha) + 1,\\
\beta &+& \lambda &\coloneqq & \sup_{\alpha < \lambda} \beta + \alpha &~ ~\text{for limit ordinals $\lambda$} \beta &+& \lambda &\coloneqq & \sup_{\alpha < \lambda} \beta + \alpha &~ ~\text{for limit ordinals $\lambda$}
\end{IEEEeqnarray*} \end{IEEEeqnarray*}
(Recall that $\alpha + 1 = \alpha \cup \{\alpha\}$ (Recall that $\alpha + 1 = \alpha \cup \{\alpha\}$

View file

@ -100,12 +100,13 @@
\item Consider $F\colon \bigcup_{i \in I}(\kappa_i \times \{i\}) \to \bigtimes_{i \in I} \lambda_i$. Want: $F$ is not surjective. \item Consider $F\colon \bigcup_{i \in I}(\kappa_i \times \{i\}) \to \bigtimes_{i \in I} \lambda_i$. Want: $F$ is not surjective.
\item Let $\xi_i$ minimal such that $\underbrace{F((\eta,i))(i)}_{\in \lambda_i} \neq \xi$ \item Let $\xi_i$ minimal such that $\underbrace{F((\eta,i))(i)}_{\in \lambda_i} \neq \xi$
for all $\eta < \kappa_i$. for all $\eta < \kappa_i$.
\item $(i \mapsto \eta_i) \not\in \ran(F)$. \item $(i \mapsto \xi_i) \not\in \ran(F)$.
\end{itemize} \end{itemize}
} }
\end{proof} \end{proof}
\begin{corollary} \begin{corollary}
\label{cor:cfpow}
For infinite cardinals $\kappa$, For infinite cardinals $\kappa$,
it is $\cf(2^{\kappa}) > \kappa$. it is $\cf(2^{\kappa}) > \kappa$.
\end{corollary} \end{corollary}
@ -155,7 +156,7 @@
The next goal is to show the following: The next goal is to show the following:
(However the method might be more interesting than the result) (However the method might be more interesting than the result)
\begin{theorem}[Silver] \begin{theorem}[Silver]
\yalabel{Silver's Theorem}{Silver}{thm:silver1} \yalabel{Silver's Theorem (case of $\aleph_{\omega_1}$)}{Silver ($\aleph_{\omega_{1}}$)}{thm:silver1}
If $2^{\aleph_\alpha} = \aleph_{\alpha + 1}$ If $2^{\aleph_\alpha} = \aleph_{\alpha + 1}$
for all $\alpha < \omega_1$, for all $\alpha < \omega_1$,
then $2^{\aleph_{\omega_1}} = \aleph_{\omega_1 + 1}$. then $2^{\aleph_{\omega_1}} = \aleph_{\omega_1 + 1}$.

View file

@ -151,7 +151,8 @@ We have shown (assuming \AxC to choose contained clubs):
is defined to be is defined to be
\[ \[
\diagi_{\beta < \alpha} A_{\beta} \coloneqq \diagi_{\beta < \alpha} A_{\beta} \coloneqq
\{\xi < \alpha : \xi \in \bigcap \{A_{\beta} : \beta < \xi\} \}. \{\xi < \alpha : \xi \in \bigcap \{A_{\beta} : \beta < \xi\} \}
= \bigcap_{\beta < \alpha} ([0,\beta] \cup A_\beta)
\] \]
\end{definition} \end{definition}
\begin{lemma} \begin{lemma}

View file

@ -35,7 +35,7 @@
Suppose otherwise. Suppose otherwise.
Then for every $\nu$ there exists a club $C_\nu$ Then for every $\nu$ there exists a club $C_\nu$
such that $S_\nu \cap C_\nu = \emptyset$.\footnote{Here we use \AxC to choose the $C_\nu$ uniformly.} such that $S_\nu \cap C_\nu = \emptyset$.\footnote{Here we use \AxC to choose the $C_\nu$ uniformly.}
Let $C = \diagi_{\nu < \alpha} C_\nu$. Let $C = \diagi_{\nu < \kappa} C_\nu$.
By \yaref{lem:diagiclub} $C$ is a club. By \yaref{lem:diagiclub} $C$ is a club.
So we may pick some $\alpha \in C \cap S$. So we may pick some $\alpha \in C \cap S$.
In particular $\alpha \in C_\nu$ for all $\nu < \alpha$. In particular $\alpha \in C_\nu$ for all $\nu < \alpha$.
@ -46,8 +46,8 @@
\begin{itemize} \begin{itemize}
\item For $\nu < \kappa$ set $S_\nu \coloneqq \{\alpha \in S : f(\alpha) = \nu\}$. \item For $\nu < \kappa$ set $S_\nu \coloneqq \{\alpha \in S : f(\alpha) = \nu\}$.
\item Suppose none of the $S_\nu$ is stationary, \item Suppose none of the $S_\nu$ is stationary,
i.e.~ $\forall \nu < \kappa.~\exists C_\nu \text{ club}.~C_\nu \cap S_\nu = \neq$. i.e.~ $\forall \nu < \kappa.~\exists C_\nu \text{ club}.~C_\nu \cap S_\nu = \emptyset$.
\item $\diagi_{\nu < \alpha} C_\nu$ is club. \item $\diagi_{\nu < \kappa} C_\nu$ is club.
\item Pick $\alpha \in C \cap S$. \item Pick $\alpha \in C \cap S$.
But then $\forall \nu < \alpha.~\alpha \in C_\nu$, i.e.~$f(\alpha) \ge \alpha$. But then $\forall \nu < \alpha.~\alpha \in C_\nu$, i.e.~$f(\alpha) \ge \alpha$.
\end{itemize} \end{itemize}
@ -104,10 +104,9 @@ to be an elementary substructure of $V_\theta$.
then $X \prec V_{\theta}$. then $X \prec V_{\theta}$.
\end{lemma} \end{lemma}
% TODO ANKI-MARKER
Let's do a second proof of \yaref{thm:fodor}. Let's do a second proof of \yaref{thm:fodor}.
\begin{refproof}{thm:fodor} \begin{refproof}{thm:fodor}
\gist{%
Fix $\theta > \kappa$ and look at $V_{\theta}$. Fix $\theta > \kappa$ and look at $V_{\theta}$.
Fix $S \subseteq \kappa$ stationary Fix $S \subseteq \kappa$ stationary
@ -158,13 +157,13 @@ Let's do a second proof of \yaref{thm:fodor}.
Let us show that $C$ is unbounded in $\kappa$. Let us show that $C$ is unbounded in $\kappa$.
Let $\zeta < \kappa$. Let $\zeta < \kappa$.
Let us define a strictly increasing sequence Let us define a strictly increasing sequence
$ \langle \xi_n n < \omega \rangle$ $ \langle \xi_n : n < \omega \rangle$
a follows. a follows.
Set $\xi_0 \coloneqq \zeta$. Set $\xi_0 \coloneqq \zeta$.
Suppose $\xi_n$ has been chosen. Suppose $\xi_n$ has been chosen.
Look at $X_{\xi_n} \cap \kappa$. Look at $X_{\xi_n} \cap \kappa$.
Since $|X_{\xi_n} \cap \kappa| < \kappa$, Since $|X_{\xi_n} \cap \kappa| < \kappa$,
$\sup (X_{\xi_n \cap \kappa}) < \kappa$. $\sup (X_{\xi_n} \cap \kappa) < \kappa$.
Set $\xi_{n+1} \coloneqq \sup(X_{\xi_n} \cap \kappa) + 1$. Set $\xi_{n+1} \coloneqq \sup(X_{\xi_n} \cap \kappa) + 1$.
Set $\xi \coloneqq \sup_{n<\omega} \xi_n$. Set $\xi \coloneqq \sup_{n<\omega} \xi_n$.
Clearly $\zeta < \xi$. Clearly $\zeta < \xi$.
@ -234,8 +233,51 @@ Let's do a second proof of \yaref{thm:fodor}.
\] \]
Hence there is some $\eta \in X_\alpha$ with $\eta \in D$. Hence there is some $\eta \in X_\alpha$ with $\eta \in D$.
This means that This means that
$\xi < \underbrace{\eta}_{\in D} < \alpha$.. $\xi < \underbrace{\eta}_{\in D} < \alpha$.
\end{subproof} \end{subproof}
}{%
\begin{itemize}
\item Fix $S \subseteq \kappa$ stationary, $f\colon S \to \kappa$ regressive.
\item Fix $\theta > \kappa$ look at $V_\theta$,
fix Skolem functions $f_\phi$ over $V_{\theta}$ for all $\phi$.
\item Define sequence of elementary substructures $\langle X_\xi, \xi < \kappa \rangle$:
\begin{itemize}
\item $X_0$ minimal st.~closed under $f_\phi$ and $S,f \in X$ (note: countable).
\item $X_{\xi+1}$ minimal st.~$X_{\xi} \subseteq X_{\xi+1}$, closed under $f_\phi$ and
$\min(\kappa \setminus X_\xi) \in X$. (note: $|X_\xi| = |X_{\xi + 1}|$)
\item $X_{\lambda} \coloneqq \bigcup_{\xi < \lambda} X_\xi$
(note: cardinality may increase)
\end{itemize}
\item Note: strictly increasing, $|X_\xi| < \kappa$, $\xi \subseteq X_\xi$.
\item $C\coloneqq \{\xi < \kappa: X_\xi \cap \kappa = \xi\}$ is club:
\begin{itemize}
\item clearly closed.
\item unbounded in $\kappa$:
\begin{itemize}
\item Take $\zeta < \kappa$, Define $\langle \xi_n : n < \omega \rangle$ by
$\xi_0 \coloneqq \zeta$, $\xi_{n+1} \coloneqq \sup(X_{\xi_n} \cap \kappa)$
($ < \kappa$), $\xi \coloneqq \sup \xi_n < \kappa$.
\item $X_\xi \cap \kappa = \xi$, i.e.~$\xi \in C$:
\begin{itemize}
\item $\eta < \xi \implies \exists n.~\eta < \xi_n \implies \eta \in \xi_n \subseteq X_{\xi_n} \subseteq X_\xi$ $\implies \xi \subseteq X_\xi \cap \kappa$.
\item $\eta \in X_\xi \cap \kappa \implies \exists n.~\eta \in X_{\xi_n} \implies
\eta < \xi_{n+1} < \xi \implies X_\xi \cap \kappa \subseteq \xi$.
\end{itemize}
\end{itemize}
\end{itemize}
\item Take $\alpha \in S \cap C$, $\nu \coloneqq f(\alpha) < \alpha$,
$T \coloneqq \{\xi \in S : f(\xi) = \nu\}$.
($T \in X_\alpha$, since it can be defined from $S, f, \nu$)
\item $T$ is stationary:
Suppose $D \cap T = \emptyset$ for a club $D$.
$X_\alpha \prec V_\theta \implies$ find such $D \in X_\alpha$.
\item Clearly $\alpha \in T$. Want ($\lightning$) $\alpha \in D$:
\begin{itemize}
\item Suffices $\sup (D \cap \alpha) = \alpha$. Let $\xi < \alpha$.
\item $D$ unbounded, so $V_\theta \models \exists \eta > \xi : \eta \in D$
\item $\implies X_\alpha \models \exists \eta > \xi . ~\eta \in D$
\item $\implies \exists \eta > \xi. ~\eta \in D \land \underbrace{\eta < \alpha}_{\in X_\alpha}$.
\end{itemize}
\end{itemize}
}
\end{refproof} \end{refproof}

View file

@ -1,5 +1,8 @@
\lecture{16}{2023-12-11}{} \lecture{16}{2023-12-11}{}
% TODO ANKI-MARKER
Recall \yaref{thm:fodor}. Recall \yaref{thm:fodor}.
\begin{question} \begin{question}
What happens if $S$ is nonstationary? What happens if $S$ is nonstationary?
@ -8,6 +11,7 @@ Let $S \subseteq \kappa$ be nonstationary,
$\kappa$ uncounable and regular. $\kappa$ uncounable and regular.
Then there is a club $C \subseteq \kappa$ Then there is a club $C \subseteq \kappa$
with $C \cap S = \emptyset$. with $C \cap S = \emptyset$.
\gist{%
Let us define $f\colon S \to \kappa$ in the following way: Let us define $f\colon S \to \kappa$ in the following way:
If $\alpha \in S$ and $C \cap \alpha \neq \emptyset$, If $\alpha \in S$ and $C \cap \alpha \neq \emptyset$,
@ -29,13 +33,18 @@ where $\gamma' = \min(C \setminus (\gamma + 1))$.
Thus for all $\gamma$, Thus for all $\gamma$,
there is only an interval of ordinals $\alpha \in S$ there is only an interval of ordinals $\alpha \in S$
where $f(\alpha) = \gamma$. where $f(\alpha) = \gamma$.
}{%
Consider $S\setminus \{0\} \ni \alpha \mapsto \underbrace{\max(C \cap \alpha)}_{< \alpha}$
(here $\max (\emptyset) \coloneqq 0$).
}
\todo{Move this to the definition of filter} \gist{%
Recall that $F \subseteq \cP(\kappa)$ is a filter if Recall that $F \subseteq \cP(\kappa)$ is a filter if
$X,Y \in F \implies X \cap Y \in F$, $X,Y \in F \implies X \cap Y \in F$,
$X \in X, X \subseteq Y \subseteq \kappa \implies Y \in F$ $X \in X, X \subseteq Y \subseteq \kappa \implies Y \in F$
and $\emptyset \not\in F, \kappa \in F$. and $\emptyset \not\in F, \kappa \in F$.
\todo{Move this to the definition of filter?}
}{}
\begin{definition} \begin{definition}
A filter $F$ is an \vocab{ultrafilter} A filter $F$ is an \vocab{ultrafilter}
iff for all $X \subseteq \kappa$ iff for all $X \subseteq \kappa$
@ -69,7 +78,7 @@ So neither $S_0$ nor $S_1 \subseteq \kappa \setminus S_0$ contains a club.
For $\kappa < \aleph_1$ For $\kappa < \aleph_1$
this argument does not work, since there is only this argument does not work, since there is only
on cofinality. one cofinality.
\begin{theorem}[Solovay] \begin{theorem}[Solovay]
\yalabel{Solovay's Theorem}{Solovay}{thm:solovay} \yalabel{Solovay's Theorem}{Solovay}{thm:solovay}
@ -94,9 +103,11 @@ on cofinality.
\begin{refproof}{thm:solovay}% \begin{refproof}{thm:solovay}%
\gist{%
%\footnote{``This is one of the arguments where it is certainly %\footnote{``This is one of the arguments where it is certainly
% worth it to look at it again''} % worth it to look at it again''}
% TODO: Look at this again and think about it. % TODO: Look at this again and think about it.
% TODO TODO TODO
We will only proof this for $\aleph_1$. We will only proof this for $\aleph_1$.
Fix $S \subseteq \aleph_1$ stationary. Fix $S \subseteq \aleph_1$ stationary.
@ -208,15 +219,54 @@ on cofinality.
\] \]
Then $\langle S_i : i < \omega_1 \rangle$ is Then $\langle S_i : i < \omega_1 \rangle$ is
as desired. as desired.
}{%
(Proof for $\aleph_1$)
Fix $S \subseteq \aleph_1$ stationary.
$\alpha \in (0, \omega_1)$ is successor ordinal or limit with $\cf(\alpha) = \omega_1$.
\begin{itemize}
\item $S^\ast \coloneqq \{\alpha \in S \setminus \{0\} : \alpha \text{ limit ordinal}\}$.
Still stationary:
$C$ club $\implies D = \{\alpha \in C \setminus \{0\} : \alpha \text{ limit}\}$ club.
\item For all $\alpha \in S^\ast$ choose $\langle \gamma^{\alpha}_n : n < \omega \rangle$ cofinal.
\item $\exists n < \omega.~\forall \delta < \omega_1: \{\alpha \in S^\ast : \gamma^{\alpha}_n > \delta\}$
stationary:
\begin{itemize}
% TODO THINK!
% TODO TODO TODO
\item Otherwise $\forall n < \omega.~\exists \delta.~\{\alpha \in S^\ast : \gamma^{\alpha}_n > \delta\} $ nonstationary.
\item $\delta_n\coloneqq $ least such $\delta$,
$C_n$ club s.t.~$C_n \cap \{\alpha \in S^\ast : \gamma^{\alpha}_n > \delta_n\} = \emptyset$.
(i.e.~$\alpha \in S^\ast \cap C_n \implies \gamma_n^{\alpha} \le \delta_n$).
\item $\delta^\ast \coloneqq \sup_{n < \omega} \delta_n$, $C \coloneqq \bigcap_{n < \omega} C_n$
is club. $\alpha \in S^\ast \cap C \implies \forall n.~\gamma_n^\alpha \le \delta^\ast$.
\item $C' \coloneqq C \setminus (\delta^\ast + 1)$ still club.
\item Pick $\alpha \in S^\ast \cap C'$.
But $\gamma_n^{\alpha} < \delta^\ast$ is cofinal $\lightning$.
\end{itemize}
\item Consider $f\colon S^\ast \to \omega_1, \alpha \mapsto \gamma_n^{\alpha}$ (note: regressive)
\item Define $\langle \delta_i : i < \omega_1 \rangle$ strictly increasing:
\begin{itemize}
\item $\delta_0 \coloneqq 0$,
\item $\delta'_{i} \coloneqq (\sup_{j < i} \delta_i) + 1$,
$\{\alpha \in S^\ast : \gamma_n^\alpha > \delta'_i\}$
is stationary $\overset{\yaref{thm:fodor}}{\implies} $ $\exists$ stationary $T_i \subseteq S^\ast$
s.t.~$f''T_i = \{\delta_i\} $ constant.
\end{itemize}
\item $T_i$ are disjoint: $i \neq j \implies \underbrace{f''T_i}_{\{\delta_i\} } \cap \underbrace{f''T_j}_{\{\delta_j\}} = \emptyset$.
\item $S_0 \coloneqq T_0 \cup \left( S \setminus \bigcup_{j > 0} T_j \right)$, $S_i \coloneqq T_i$.
\end{itemize}
}
\end{refproof} \end{refproof}
\gist{%
We now want to do another application of \yaref{thm:fodor}. We now want to do another application of \yaref{thm:fodor}.
Recall that $2^{\kappa} > \kappa$, in fact $\cf(2^{\kappa}) > \kappa$ Recall that $2^{\kappa} > \kappa$, in fact $\cf(2^{\kappa}) > \kappa$
by \yaref{thm:koenig}. by \yaref{thm:koenig}
(cf.~\yaref{cor:cfpow}).
Trivially, if $\kappa \le \lambda$ then $2^{\kappa} \le 2^{\lambda}$. Trivially, if $\kappa \le \lambda$ then $2^{\kappa} \le 2^{\lambda}$.
This is in some sense the only thing we can prove about successor cardinals. This is in some sense the only thing we can prove about successor cardinals.
However we can say something about singular cardinals: However we can say something about singular cardinals:
}{}
\begin{theorem}[Silver] \begin{theorem}[Silver]
\yalabel{Silver's Theorem}{Silver}{thm:silver} \yalabel{Silver's Theorem}{Silver}{thm:silver}
@ -231,6 +281,7 @@ However we can say something about singular cardinals:
is the statement is the statement
that $2^{\lambda} = \lambda^+$ holds for all infinite cardinals $\lambda$, that $2^{\lambda} = \lambda^+$ holds for all infinite cardinals $\lambda$,
\end{definition} \end{definition}
\gist{%
Recall that $\CH$ says that $2^{\aleph_0} = \aleph_1$. Recall that $\CH$ says that $2^{\aleph_0} = \aleph_1$.
So $\GCH \implies \CH$. So $\GCH \implies \CH$.
@ -239,3 +290,4 @@ then it is true at $\kappa$.
The proof of \yalabel{thm:silver} is quite elementary, The proof of \yalabel{thm:silver} is quite elementary,
so we will do it now, but the statement can only be fully appreciated later. so we will do it now, but the statement can only be fully appreciated later.
}{}

View file

@ -1,18 +1,24 @@
\lecture{17}{2023-12-14}{Silver's Theorem} \lecture{17}{2023-12-14}{Silver's Theorem}
We now want to prove \yaref{thm:silver}. We now want to prove \yaref{thm:silver}.
More generally, if $\kappa$ is a singular cardinal of uncountable cofinality % More generally, if $\kappa$ is a singular cardinal of uncountable cofinality
such that $2^{\lambda} = \lambda^+$ for all $\lambda < \kappa$, % such that $2^{\lambda} = \lambda^+$ for all $\lambda < \kappa$,
then $2^{\kappa} = \kappa^+$. % then $2^{\kappa} = \kappa^+$.
\gist{%
\begin{remark} \begin{remark}
The hypothesis of \yaref{thm:silver} The hypothesis of \yaref{thm:silver}
is consistent with $\ZFC$. is consistent with $\ZFC$.
\end{remark} \end{remark}
}{}
We will only proof \yaref{thm:silver} in the special case that $\kappa = \aleph_{\omega_1}$. We will only proof
\gist{%
\yaref{thm:silver} in the special case that $\kappa = \aleph_{\omega_1}$
(see \yaref{thm:silver1}).
The general proof differs only in notation. The general proof differs only in notation.
}{\yaref{thm:silver1}.}
\gist{%
\begin{remark} \begin{remark}
It is important that the cofinality is uncountable. It is important that the cofinality is uncountable.
For example it is consistent For example it is consistent
@ -20,8 +26,10 @@ The general proof differs only in notation.
$2^{\aleph_n} = \aleph_{n+1}$ for all $n < \omega$ $2^{\aleph_n} = \aleph_{n+1}$ for all $n < \omega$
but at the same time $2^{\aleph_{\omega}} = \aleph_{\omega + 2}$. but at the same time $2^{\aleph_{\omega}} = \aleph_{\omega + 2}$.
\end{remark} \end{remark}
}{}
\begin{refproof}{thm:silver} \begin{refproof}{thm:silver1}
\gist{%
We need to count the number of $X \subseteq \aleph_{\omega_1}.$ We need to count the number of $X \subseteq \aleph_{\omega_1}.$
Let us fix $\langle f_\lambda : \lambda < \kappa \text{ an infinite cardinal} \rangle$ Let us fix $\langle f_\lambda : \lambda < \kappa \text{ an infinite cardinal} \rangle$
such that $f_{\lambda}\colon \cP(\lambda) \to \lambda^+$ such that $f_{\lambda}\colon \cP(\lambda) \to \lambda^+$
@ -203,18 +211,58 @@ The general proof differs only in notation.
The set The set
\[ \[
\{Y \subseteq \aleph_{\omega_1} : \exists i < \aleph_{\omega_1 + 1} .~Y \le X_i\} P \coloneqq \{Y \subseteq \aleph_{\omega_1} : \exists i < \aleph_{\omega_1 + 1} .~Y \le X_i\}
= \bigcup_{i < \aleph_{\omega_1 + 1}} \{Y \subseteq \aleph_{\omega_1} : Y \le X_i\} = \bigcup_{i < \aleph_{\omega_1 + 1}} \{Y \subseteq \aleph_{\omega_1} : Y \le X_i\}
\] \]
has size $\le \aleph_{\omega_1 + 1}$ has size $\le \aleph_{\omega_1 + 1}$
(in fact the size is exactly $\aleph_{\omega_1 + 1}$). (in fact the size is exactly $\aleph_{\omega_1 + 1}$).
But On the other hand
\[ $P = \cP(\aleph_{\omega_1})$
\{Y \subseteq \aleph_{\omega_1} : \exists i < \aleph_{\omega_1 + 1} .~Y \le X_i\} = \cP(\aleph_{\omega_1 + 1}) because if $X \subseteq \aleph_{\omega_1}$
\]
because if $X \subseteq \aleph_{\omega_1 + 1}$
is such that $X \nleq X_i$ for all $i < \aleph_{\omega_1 + 1}$, is such that $X \nleq X_i$ for all $i < \aleph_{\omega_1 + 1}$,
then $X_i \le X$ for all $i < \aleph_{\omega_1 + 1}$, then $X_i \le X$ for all $i < \aleph_{\omega_1 + 1}$,
so such a set $X$ does not exist by \yaref{thm:silver:p:c3}. so such a set $X$ does not exist by \yaref{thm:silver:p:c3}.
}{Need to count $X \subseteq \aleph_{ \omega_{1}}$.
\begin{itemize}
\item Fix bijections $f_\lambda\colon 2^{\lambda} \to \lambda^+$
for all infinite cardinals $\lambda < \kappa$.
\item For $X \subseteq \aleph_{ \omega_1}$ define $f_X\colon \omega_1 \to \aleph_{ \omega_1},
\alpha \mapsto f_{\aleph_\alpha}(X \cap \aleph_\alpha)$.
\item (1) $X \neq Y \iff f_X \neq f_Y$:
\begin{itemize}
\item $X \neq Y \iff \exists \alpha.~X \cap \aleph_\alpha \neq Y \cap \aleph_\alpha \iff \exists \alpha.~f_X(\alpha) \neq f_Y(\alpha)$.
\end{itemize}
\item $X \le Y :\iff \{\alpha < \omega_1 : f_X(\alpha) \le f_Y(\alpha)\}$ stationary.
\item (2) $X\le Y \lor Y \le X$:
Suppose $X \not\le Y, Y \not\le X$. Choose witnessing clubs $C, D$.
$C \cap D$ is club, but then $f_X(\alpha) \le f_Y(\alpha)$ or
$f_X(\alpha) \ge f_Y(\alpha)$ for $\alpha \in C \cap D$.
\item (3) $X \subseteq \aleph_{ \omega_1}$, then $|\underbrace{\{Y \subseteq \aleph_{ \omega_1} : Y \le X\}}_{A}| \le \aleph_{ \omega_1}$
\begin{itemize}
\item Suppose $|A| \ge \aleph_{ \omega_1 + 1}$.
\item $S_Y \coloneqq \{\alpha : f_Y(\alpha) \le f_X(\alpha)\}$ stationary for all $Y \in A$.
$2^{\aleph_1} = \aleph_2 \implies$ at most $\aleph_2$ such $S_Y$.
\item If $\forall S \in \cP( \omega_1).~ |\underbrace{\{Y \in A : S_Y = S\}}_{A_S}| < \aleph_{ \omega_1 + 1}$,
then $|A| \le \aleph_2 \cdot <\aleph_{ \omega_1 + 1}$ $\lightning$ $\aleph_{ \omega_1 + 1}$ regular.
\item So $\exists S \in \omega_1.~|A_S| = \aleph_{ \omega_1 + 1 + 1}$.
% TODO TODO TODO hier weiter!
\end{itemize}
\item Define sequence $\langle X_i : i < \aleph_{ \omega_1 + 1} \rangle$
of subsets of $\aleph_{\omega_1}$:
\begin{itemize}
\item Consider $\{Y \subseteq \aleph_{ \omega_1} : \exists j < i.~Y \le X_j\}$
(cardinality $ \le \aleph_{ \omega_1}$),
Take $X_i \subseteq \aleph_{ \omega_1}$
such that $X_i \subseteq \aleph_{ \omega_1}$
such that $X_i \not\le X_j$ for all $j < i$.
\item $P \coloneqq \{Y \subseteq \aleph_{ \omega_1} : \exists i < \aleph_{ \omega_1 + 1}.~Y \le X_i\}$
\end{itemize}
\item $|P| \overset{\text{in fact } =}{\le} \aleph_{ \omega_1 + 1} \aleph_{ \omega_1} = \aleph_{ \omega_1 + 1}$ by (3).
\item $X \in \cP(\aleph_{ \omega_1}) \setminus M \implies \forall i < \aleph_{ \omega_1 + 1}.~X_i \le X$
$\lightning$ (3).
Thus $P = \cP(\aleph_{ \omega_1})$.
\end{itemize}
}
\end{refproof} \end{refproof}